document.write( "Question 1209919: Let x_1, x_2, \dots, x_n be real numbers. If
\n" ); document.write( "x_1^2 + 2x_2^2 + \dots + nx_n^2 = 1,
\n" ); document.write( "then find the maximum value of (x_1 + x_2/2 + \dots + x_n/n)^2, in terms of n.
\n" ); document.write( "

Algebra.Com's Answer #851057 by CPhill(1959)\"\" \"About 
You can put this solution on YOUR website!
Let $a_k = x_k \sqrt{k}$ for $k = 1, 2, \dots, n$.
\n" ); document.write( "Then the given condition becomes
\n" ); document.write( "$$\sum_{k=1}^n x_k^2 k = \sum_{k=1}^n a_k^2 = 1$$\r
\n" ); document.write( "\n" ); document.write( "We want to find the maximum value of
\n" ); document.write( "$$S = \left( \sum_{k=1}^n \frac{x_k}{k} \right)^2 = \left( \sum_{k=1}^n \frac{a_k}{k\sqrt{k}} \right)^2$$\r
\n" ); document.write( "\n" ); document.write( "By the Cauchy-Schwarz inequality, we have
\n" ); document.write( "$$\left( \sum_{k=1}^n \frac{a_k}{k\sqrt{k}} \right)^2 \le \left( \sum_{k=1}^n a_k^2 \right) \left( \sum_{k=1}^n \frac{1}{k^3} \right)$$
\n" ); document.write( "Since $\sum_{k=1}^n a_k^2 = 1$, we have
\n" ); document.write( "$$S \le \sum_{k=1}^n \frac{1}{k^3}$$\r
\n" ); document.write( "\n" ); document.write( "The maximum value of $\left( \sum_{k=1}^n \frac{x_k}{k} \right)^2$ is $\sum_{k=1}^n \frac{1}{k^3}$.\r
\n" ); document.write( "\n" ); document.write( "To achieve equality in Cauchy-Schwarz inequality, we need
\n" ); document.write( "$$\frac{a_1}{1\sqrt{1}} = \frac{a_2}{2\sqrt{2}} = \dots = \frac{a_n}{n\sqrt{n}} = c$$
\n" ); document.write( "where $c$ is a constant.
\n" ); document.write( "Then $a_k = ck\sqrt{k}$, so $x_k \sqrt{k} = ck\sqrt{k}$, which means $x_k = ck$.
\n" ); document.write( "Substituting into the given condition, we have
\n" ); document.write( "$$\sum_{k=1}^n kx_k^2 = \sum_{k=1}^n k(ck)^2 = c^2 \sum_{k=1}^n k^3 = 1$$
\n" ); document.write( "$$c^2 = \frac{1}{\sum_{k=1}^n k^3}$$
\n" ); document.write( "$$c = \frac{1}{\sqrt{\sum_{k=1}^n k^3}}$$
\n" ); document.write( "Then $x_k = \frac{k}{\sqrt{\sum_{k=1}^n k^3}}$.\r
\n" ); document.write( "\n" ); document.write( "Therefore, the maximum value of $\left( \sum_{k=1}^n \frac{x_k}{k} \right)^2$ is $\sum_{k=1}^n \frac{1}{k^3}$.\r
\n" ); document.write( "\n" ); document.write( "Final Answer: The final answer is $\boxed{\sum_{k=1}^n \frac{1}{k^3}}$
\n" ); document.write( "
\n" ); document.write( "
\n" );